LSAT and Law School Admissions Forum

Get expert LSAT preparation and law school admissions advice from PowerScore Test Preparation.

 Administrator
PowerScore Staff
  • PowerScore Staff
  • Posts: 8919
  • Joined: Feb 02, 2011
|
#27397
Complete Question Explanation

Resolve the Paradox. The correct answer choice is (B)

Here we are presented with information on a new type of seismic shock—a displacement pulse—thought to be part of every earthquake. In spite of the fact that computer models have shown high-rise buildings to be very vulnerable to these shocks, recent powerful earthquakes did little significant damage to these tall buildings.

The answer choice which resolves this paradox will be the one which is consistent with both the dire computer predictions and the real-world recent experience of limited damage to the high-rise buildings.

Answer choice (A): This choice fails to resolve the paradox, and in fact seems to broaden it: If displacement pulses travel farther than other types of seismic shock, why are they not damaging the high-rise buildings as predicted. Since this answer choice does not help to explain the discrepancy from the stimulus, it is incorrect.

Answer choice (B): This is the correct answer choice. This answer choice helps to explain how it could be that the buildings don’t seem as vulnerable as predicted by computer models. If predictions based on computer models often fail in the real world, then this would help to explain the divergence between the computer model predictions and the real world effects of displacement pulses on high-rise buildings.

Answer choice (C): So displacement pulses were always part of earthquakes. This does not help to explain why high-rise buildings are apparently not as vulnerable to these pulses as predicted by computer models.

Answer choice (D): This answer choice, which deals with low and medium intensity earthquakes, does not help to explain why the stimulus’ referenced earthquakes (“recent powerful earthquakes”) have not done significant damage to the high rise buildings.

Answer choice (E): The effectiveness of computer models to predict other types of shock is irrelevant to the question of why buildings don’t seem to respond to displacement pulses as predicted by computer models, so this answer choice resolves nothing and cannot be correct.
 a19
  • Posts: 11
  • Joined: Jul 04, 2019
|
#77575
Hi,

So my apologies. I know this is an easy question, yet I am a bit torn between why C could not also be the answer. These "recent investigations" only examined "recent powerful earthquakes," and predicted that this new element, "a displacement pulse," was only just accounted for. But this shock, is "believed to be present in ALL earthquakes."

Looking at the answers, it makes sense that if the models are wrong, the predictions would be wrong. Hence, I get B.

The only reason I see that C wouldnt work is because the stimulus said that the displacement pulse was believed to be in "all earthquakes." Essentially making it a restated premise..... ?

But even so it offers more information than "all earthquakes" in the stimulus, as it incorporates a chronological element into the premise, "that have ever occurred." This also contrasts with the recency of the investigations and the recent powerful earthquakes. Finally if this pulse has occurred in every earthquake since the beginning of time, then we would have no high-rises (we would be presuming that the models are correct.) Rather, since this pulse has occurred in every earthquake AND we know that there are standing high-rises, then we know that skyscrapers can withstand these earthquakes--potentially due to the fact that they were built to withstand earthquakes. Would this implication not also resolve the paradox as to why computer models and the actual recorded data do not match?

I feel like im overthinking this one, but even in a timed setting, I found myself thinking about these possibilities.
 Jeremy Press
PowerScore Staff
  • PowerScore Staff
  • Posts: 1000
  • Joined: Jun 12, 2017
|
#77903
Hi a19,

I think you almost implied the answer to your question at the end there: "Would this implication not also resolve the paradox as to why computer models and the actual recorded data do not match?" It wouldn't!

The problem with answer choice C is that, even under the reading you're giving it, it still doesn't fully resolve the paradox. This is because it doesn't explain why the computer models are predicting damage. It might tell us that they're wrong, but it wouldn't tell us why. Indeed, if what answer choice C says were true (and you assume, as you have, that you'd never have had high rises in the first place), the model should've accounted for that historical fact and not predicted damage, right? There would still be a mystery as to why the model predicted damage. Our Resolve the Paradox answer would still need to explain that.

Answer choice B does that: the model likely failed, because they often fail when tested in the real world!

Side note: don't incorporate answer choice B into answer choice C, and don't assess answer choice B (or C) based on the other answer. That's a problem I've seen people have on Family 2 questions like this sometimes. Evaluate each answer by itself and ask whether, by itself, it explains the full puzzle of the stimulus.

I hope this helps!

Jeremy
 cleocleozuo
  • Posts: 21
  • Joined: Jun 02, 2020
|
#85191
Hi,

I have a question about B though... I know in Paradox questions we can not solve a "surprising" finding by attacking either side of the issue. But I feel B is actually attacking one of the sides by saying computer findings are often inaccurate. Am I mistaken?

Many thanks!
 Adam Tyson
PowerScore Staff
  • PowerScore Staff
  • Posts: 5153
  • Joined: Apr 14, 2011
|
#85209
Good question, cleocleozuo, and it raises an important distinction! When we talk about attacking the information, we are talking about claiming that the information provided is inaccurate. That's like disagreeing with a premise, something we generally don't do in LR questions that involve arguments.

In this case, attacking that side of the paradox would mean saying "actually, the computer models didn't show that high-rise buildings are vulnerable." That's denying the facts, and is the wrong way to resolve a paradox. But what answer B is doing is saying "I accept that the computers models said that high-rise buildings are vulnerable, but the computers models could have been wrong." This would be like saying that the models were based on faulty programming, or that they failed to account for all relevant factors, either of which could be a cause for the models saying something would happen that did not end up happening.

And that's the thing we are looking for in a good Resolve answer - the cause of the odd situation. How did we get actual results that varied from what the models predicted? What caused this to occur? Faulty computer models could be the cause of that discrepancy! We aren't denying what the models said, but saying that what they said could be incorrect because of something not accounted for in the stimulus.
 cleocleozuo
  • Posts: 21
  • Joined: Jun 02, 2020
|
#85239
Thank you for clearing this up, It helps a lot!

Get the most out of your LSAT Prep Plus subscription.

Analyze and track your performance with our Testing and Analytics Package.